LSAT and Law School Admissions Forum

Get expert LSAT preparation and law school admissions advice from PowerScore Test Preparation.

User avatar
 Dave Killoran
PowerScore Staff
  • PowerScore Staff
  • Posts: 5853
  • Joined: Mar 25, 2011
|
#42386
Complete Question Explanation
(The complete setup for this game can be found here: lsat/viewtopic.php?t=16021)

The correct answer choice is (E)

If K and M are assigned to different tables, one must be assigned to table 1 and the other must be assigned to table 2. Since we cannot be certain which is assigned to table 1 or table 2, it is best to display this situation as a dual option:
J97_Game_#1_#3_diagram 1.png
Since the LV block must be seated at either table 1 or table 2, it follows that table 1 or table 2 must have three sponsors seated at it. Therefore, table 3 can only have two sponsors seated at it and answer choice (E) is proven correct. Answer choice (D) is incorrect since it is possible for three sponsors to sit at table 1. Answer choices (A) and (C) are both incorrect since the LV block can sit at either table 1 or table 2, as can K or M. Thus, although many combinations of K, M, and the LV block are possible, none must occur. Answer choice (B) is incorrect since L and Q can never sit together given the condition in the question stem.
You do not have the required permissions to view the files attached to this post.

Get the most out of your LSAT Prep Plus subscription.

Analyze and track your performance with our Testing and Analytics Package.